Difference between revisions of "User talk:Bobthesmartypants/Sandbox"

(Solution)
(Solution)
Line 142: Line 142:
 
label("$1$",(sqrt(3),0.5),dir(0));
 
label("$1$",(sqrt(3),0.5),dir(0));
 
label("$\frac{1}{\tan\varphi}$",(sqrt(3)/2,0),dir(-90));
 
label("$\frac{1}{\tan\varphi}$",(sqrt(3)/2,0),dir(-90));
 +
</asy>
 +
 +
<asy>
 +
import olympiad;
 +
 +
 +
size(300,300);
 +
draw((0,0)--10*dir(11.42));
 +
draw(Circle((10,1),1));
 +
dot((0,0));
 +
draw((-1,0)--(12,0),grey);
 +
draw((0,0)--(0,3),grey);
 +
draw(anglemark(10*dir(11.42),(0,0),(0,1)));
 +
label("$\theta$",0.3*dir(50.71),dir(50.71));
 +
draw((0,0)--(10,1),linetype("8 8"));
 +
draw(10*dir(11.42)--(10,1)--(10,0),linetype("8 8"));
 +
label("$1$",(10,0.5),dir(0));
 +
label("$10$",(5,0),dir(-90));
 +
label("$\varphi$",3.4*dir(2.855),dir(2.855));
 +
markscalefactor=0.4;
 +
draw(anglemark((1,0),(0,0),dir(5.71)));
 
</asy>
 
</asy>

Revision as of 14:51, 27 April 2014

Bobthesmartypants's Sandbox

Solution 1

[asy] path Q; Q=(0,0)--(1,2)--(5,2)--(4,0)--cycle; draw(Q); draw((0,0)--(1.5,1)); label("D",(0,0),S); draw((1,2)--(1.5,1)); label("A",(1,2),N); draw((5,2)--(1.5,1)); label("B",(5,2),N); draw((4,0)--(1.5,1)); label("C",(4,0),S); draw((2,0)--(1.5,1),linetype("8 8")); label("E",(2,0),S); draw((2/3,4/3)--(1.5,1),linetype("8 8")); label("F",(2/3,4/3),W); label("P",(1.5,1),NNE); [/asy]

First, continue $\overline{AP}$ to hit $\overline{CD}$ at $E$. Also continue $\overline{CP}$ to hit $\overline{AD}$ at $F$.

We have that $\angle PAB=\angle PCB$. Because $\overline{AB}\parallel\overline{CD}$, we have $\angle PAB=\angle PED$.

Similarly, because $\overline{AD}\parallel\overline{BC}$, we have $\angle PCB=\angle PFD$.

Therefore, $\angle PAB=\angle PED=\angle PCB=\angle PFD$.

We also have that $\angle ADC=\angle ABC$ because $ABCD$ is a parallelogram, and $\angle APC=\angle FPE$.

Therefore, $ABCP\sim FDEP$. This means that $\dfrac{FD}{AB}=\dfrac{FP}{AP}=\dfrac{DP}{BP}$, so $\Delta ABP\sim\Delta FDP$.

Therefore, $\angle PBA=\angle PDA$. $\Box$


Solution 2

Note that $\dfrac{1}{n}$ is rational and $n$ is not divisible by $2$ nor $5$ because $n>11$.

This means the decimal representation of $\dfrac{1}{n}$ is a repeating decimal.

Let us set $a_1a_2\cdots a_x$ as the block that repeats in the repeating decimal: $\dfrac{1}{n}=0.\overline{a_1a_2\cdots a_x}$.

($a_1a_2\cdots a_x$ written without the overline used to signify one number so won't confuse with notation for repeating decimal)

The fractional representation of this repeating decimal would be $\dfrac{1}{n}=\dfrac{a_1a_2\cdots a_x}{10^x-1}$.

Taking the reciprocal of both sides you get $n=\dfrac{10^x-1}{a_1a_2\cdots a_x}$.

Multiplying both sides by $a_1a_2\cdots a_n$ gives $n(a_1a_2\cdots a_x)=10^x-1$.

Since $10^x-1=9\times \underbrace{111\cdots 111}_{x\text{ times}}$ we divide $9$ on both sides of the equation to get $\dfrac{n(a_1a_2\cdots a_x)}{9}=\underbrace{111\cdots 111}_{x\text{ times}}$.

Because $n$ is not divisible by $3$ (therefore $9$) since $n>11$ and $n$ is prime, it follows that $n|\underbrace{111\cdots 111}_{x\text{ times}}$. $\Box$

Picture 1

[asy]draw(Circle((1,1),2)); draw(Circle((sqrt(2),sqrt(3)/2),1)); dot((8/5,2/5)); dot((1,1)); draw((1,1)--(8/5,2/5),linetype("8 8")); label("a",(6/5,7/10),SSW); draw((8/5,2/5)--(12/5,-2/5),linetype("8 8")); label("b",(2,0),SSW); [/asy] \[\text{Find the probability that }b>a \text{.}\]

Picture 2

[asy] for (int i=0;i<6;i=i+1){ draw(dir(60*i)--dir(60*i+60)); } draw(dir(120)--(dir(0)+dir(-60))/2); draw(dir(180)--(dir(60)+dir(0))/2); fill(dir(120)--dir(180)--intersectionpoint(dir(120)--(dir(0)+dir(-60))/2,dir(180)--(dir(60)+dir(0))/2)--cycle,grey); fill((dir(0)+dir(-60))/2--dir(0)--(dir(60)+dir(0))/2--intersectionpoint(dir(120)--(dir(0)+dir(-60))/2,dir(180)--(dir(60)+dir(0))/2)--cycle,grey); [/asy] \[\text{Prove the shaded areas are equal.}\] [asy] for(int i = 0; i < 8; ++i){   for(int j = 0; j < 8; ++j){     filldraw((i,j)--(i+1,j)--(i+1,j+1)--(i,j+1)--cycle,black);   } } for(int i = 1; i < 7; ++i){   filldraw((i,7-i)--(i+1,7-i)--(i+1,8-i)--(i,8-i)--cycle,white); } for(int i = 0; i < 5; ++i){   filldraw((i,4-i)--(i+1,4-i)--(i+1,5-i)--(i,5-i)--cycle,white); }  for(int i = 0; i < 5; ++i){   filldraw((8-i,4+i)--(7-i,4+i)--(7-i,3+i)--(8-i,3+i)--cycle,white); }  filldraw((1,0)--(2,0)--(2,1)--(1,1)--cycle,white); filldraw((0,1)--(0,2)--(1,2)--(1,1)--cycle,white); filldraw((7,8)--(6,8)--(6,7)--(7,7)--cycle,white); filldraw((8,7)--(8,6)--(7,6)--(7,7)--cycle,white); [/asy]

physics problem

[asy]size(100,100); draw((0,0)--(0,10)); draw(Circle((1,1),1)); dot((0,0)); draw(9.5*dir(80)..9.5*dir(70)..9.5*dir(60),EndArrow()); label("A",(-1,5),dir(180));[/asy]

[asy] size(85,85); draw((0,0)--10*dir(60)); draw(Circle((sqrt(3),1),1)); dot((0,0)); draw(9.5*dir(50)..9.5*dir(40)..9.5*dir(30),EndArrow()); label("B",(0,5),dir(0)); [/asy]

[asy] size(110,110); draw((0,0)--10*dir(11.42)); draw(Circle((10,1),1)); dot((0,0)); label("C",(5,1.5),dir(90)); [/asy]

Solution

[asy] import olympiad;   size(350,350); draw((0,0)--10*dir(60)); draw(Circle((sqrt(3),1),1)); dot((0,0)); draw((-1,0)--(7,0),grey); draw((0,0)--(sqrt(3),1),linetype("8 8")); draw((0,0)--(0,5),grey); draw(sqrt(3)*dir(60)--(sqrt(3),1)--(sqrt(3),0),linetype("8 8")); draw(anglemark((1,0),(0,0),dir(30))); label("$\varphi$",0.3*dir(15),dir(15)); draw(anglemark(dir(60),(0,0),(0,1))); label("$\theta$",0.3*dir(75),dir(75)); label("$1$",(sqrt(3),0.5),dir(0)); label("$\frac{1}{\tan\varphi}$",(sqrt(3)/2,0),dir(-90)); [/asy]

[asy] import olympiad;   size(300,300); draw((0,0)--10*dir(11.42)); draw(Circle((10,1),1)); dot((0,0)); draw((-1,0)--(12,0),grey); draw((0,0)--(0,3),grey); draw(anglemark(10*dir(11.42),(0,0),(0,1))); label("$\theta$",0.3*dir(50.71),dir(50.71)); draw((0,0)--(10,1),linetype("8 8")); draw(10*dir(11.42)--(10,1)--(10,0),linetype("8 8")); label("$1$",(10,0.5),dir(0)); label("$10$",(5,0),dir(-90)); label("$\varphi$",3.4*dir(2.855),dir(2.855)); markscalefactor=0.4; draw(anglemark((1,0),(0,0),dir(5.71))); [/asy]